Toán học - Đồng nhất thức và bất đẳng thức trong tam giác

pdf62 trang | Chia sẻ: minhhong95 | Lượt xem: 769 | Lượt tải: 0download
Bạn đang xem trước 20 trang mẫu tài liệu Toán học - Đồng nhất thức và bất đẳng thức trong tam giác, để tải tài liệu gốc về máy bạn click vào nút DOWNLOAD ở trên
ĐẠI HỌC THÁI NGUYÊN
TRƯỜNG ĐẠI HỌC KHOA HỌC
Nguyễn Thị Út
ĐỒNG NHẤT THỨC VÀ BẤT ĐẲNG THỨC
TRONG TAM GIÁC
SOME IDENTITIES AND INEQUALITIES
OF TRIANGLES
Chuyên ngành: PHƯƠNG PHÁP TOÁN SƠ CẤP
Mã số: 60.46.40
LUẬN VĂN THẠC SĨ TOÁN HỌC
Người hướng dẫn khoa học: PGS.TS. Đàm Văn Nhỉ
Thái Nguyên - 2012
Công trình được hoàn thành tại
Trường Đại học Khoa học - Đại học Thái Nguyên
Người hướng dẫn khoa học: PGS.TS. Đàm Văn Nhỉ
Phản biện 1: . . . . . . . . . . . . . . . . . . . . . . . . . . . . . . . . . . . . . . . . . . . . . . . . . . . . . . .
. . . . . . . . . . . . . . . . . . . . . . . . . . . . . . . . . . . . . . . . . . . . . . . . . . . . . . . . . . . . . . . . . . . .
Phản biện 2: . . . . . . . . . . . . . . . . . . . . . . . . . . . . . . . . . . . . . . . . . . . . . . . . . . . . . . .
. . . . . . . . . . . . . . . . . . . . . . . . . . . . . . . . . . . . . . . . . . . . . . . . . . . . . . . . . . . . . . . . . . . .
Luận văn sẽ được bảo vệ trước hội đồng chấm luận văn họp tại:
Trường Đại học Khoa học - Đại học Thái Nguyên
Ngày .... tháng .... năm 2012
Có thể tìm hiểu tại
Thư viện Đại học Thái Nguyên
1Mục lục
Mục lục . . . . . . . . . . . . . . . . . . . . . . . . . . . . . 1
Mở đầu . . . . . . . . . . . . . . . . . . . . . . . . . . . . . . 2
Chương 1. Kiến thức chuẩn bị 4
1.1. Bất đẳng thức qua tam thức bậc hai . . . . . . . . . . . 4
1.2. Bất đẳng thức Jensen . . . . . . . . . . . . . . . . . . . . 5
1.3. Bất đẳng thức Karamata, Schur, Muirheard . . . . . . . 8
1.4. Một vài hàm tự chọn . . . . . . . . . . . . . . . . . . . . 12
Chương 2. Một số đồng nhất thức và bất đẳng thức trong
tam giác 14
2.1. Đa thức bậc ba liên quan đến tam giác. . . . . . . . . . 14
2.2. Một số bất đẳng thức trong tam giác . . . . . . . . . . . 25
2.3. Một số bài toán nhận dạng tam giác . . . . . . . . . . . 37
Chương 3. Trình bày một số kết quả của J.Liu [8] và của
Klamkin [7]. 43
3.1. Khai thác bài toán véc tơ trong mặt phẳng . . . . . . . . 43
3.2. Trình bày lại kết quả bài báo của J.Liu . . . . . . . . . . 51
3.2.1. Một số định lý . . . . . . . . . . . . . . . . . . . 51
3.2.2. Một vài bổ đề . . . . . . . . . . . . . . . . . . . . 52
3.2.3. Chứng minh ba định lý trên . . . . . . . . . . . . 53
3.3. Trình bày bất đẳng thức của Klamkin . . . . . . . . . . 55
Kết luận . . . . . . . . . . . . . . . . . . . . . . . . . . . . . 59
Tài liệu tham khảo . . . . . . . . . . . . . . . . . . . . . . . 60
2Mở đầu
Đồng nhất thức và bất đẳng thức trong tam giác là một chuyên mục
hấp dẫn đối với những người quan tâm tới Hình sơ cấp. Đây là một mảnh
đất đã được cày xới quá nhiều qua năm tháng. Vấn đề đặt ra: Làm thế
nào để có đồng nhất thức và bất đẳng thức mới trong tam giác.
Tam giác là một hình quen thuộc đối với tất cả mọi người. Thông
thường, khi xét bài toán hình học người ta thường phải dùng đến thước
kẻ, compa và giải quyết bài toán ấy qua hình vẽ. Nhưng cách làm như
vậy rất khó phát hiện ra hệ thức mới. Chúng ta càng khó xây dựng được
bài toán với nhiều đại lượng của tam giác. Do có quá nhiều kết quả trong
tam giác nên xuất hiện câu hỏi thứ nhất: Có thể xây dựng được kết quả
mới hay không? Nhiều người sử dụng lượng giác, hình vẽ, phương pháp
diện tích,v.v... để tạo ra kết quả. Theo chúng tôi, những cách xây dựng
như vậy rất khó đưa ra hệ thức cho tam giác mà có nhiều thành phần
tham gia. Rất tự nhiên, xuất hiện câu hỏi thứ hai: Xây dựng kết quả
như thế nào? Bài toán đặt ra: Xây dựng đồng nhất thức và bất đẳng
thức trong tam giác. Với luận văn này, chúng tôi mong muốn giải quyết
được một phần nào đó thuộc bài toán trên.
Luận văn được chia ra làm ba chương.
Chương 1. Kiến thức chuẩn bị.
Chương này tập trung trình bày về một số bất đẳng thức. Nó bao gồm
các mục: Bất đẳng thức qua tam thức bậc 2, bất đẳng thức Jensen qua
hàm lồi và bất đẳng thức Muirheard, Karamata. Ngoài ra, để phát hiện
ra một số bất đẳng thức khác nữa cho tam giác chúng tôi đã chọn ra
một số hàm tương ứng với mục đích đặt ra.
Chương 2. Một số đồng nhất thức và bất đẳng thức trong
tam giác.
Đây là nội dung trọng tâm của luận văn. Nó bao gồm các mục sau: Mục
32.1 tập trung xây dựng một số đa thức bậc ba liên quan tam giác. Từ
những đa thức này ta đã có thể phát hiện ra một số đồng nhất thức
và bất đẳng thức mới trong tam giác. Mục 2.2 tập trung xây dựng và
chứng minh lại một số bất đẳng thức trong tam giác qua việc sử dụng
các kết quả ở Chương 1. Từ các kết quả đạt được chúng ta sẽ phát hiện
ra những tam giác đặc biệt với điều kiện ban đầu đặt ra ở Mục 2.3.
Chương 3. Trình bày một số kết quả của J.Liu [8] và của
Klamkin [7].
Chương này dành để trình bày việc khai thác một bài toán véc tơ trong
mặt phẳng ở Mục 3.1. Mục 3.2 trình bày lại một số kết quả của J. Liu
trong bài báo [8]. Mục 3.3 trình bày lại kết quả của Klamkin trong [7].
Luận văn này được hoàn thành với sự hướng dẫn và chỉ bảo tận tình
của PGS,TS Đàm Văn Nhỉ. Từ đáy lòng mình, em xin được bày tỏ lòng
biết ơn sâu sắc đối với sự quan tâm, động viên và sự chỉ bảo hướng dẫn
của thầy.
Em xin trân trọng cảm ơn tới các Thầy, Cô giáo trong Trường Đại
học Khoa học - Đại học Thái Nguyên, phòng Đào tạo Trường Đại học
Khoa học. Đồng thời tác giả xin gửi lời cảm ơn tới tập thể lớp Cao học
Toán K4 Trường Đại học Khoa học đã động viên giúp đỡ tôi trong quá
trình học tập và làm luận văn này.
Tuy nhiên do sự hiểu biết của bản thân, điều kiện thời gian và khuôn
khổ của luận văn thạc sĩ, nên chắc rằng trong quá trình nghiên cứu
không tránh khỏi những khiếm khuyết.Tác giả rất mong được sự chỉ dạy
và đóng góp ý kiến của các Thầy, Cô giáo và quý vị bạn đọc đóng góp
ý kiến để luận văn được hoàn thành tốt hơn.
Thái Nguyên, ngày 10 tháng 10 năm 2012
Tác giả
Nguyễn Thị Út
4Chương 1
Kiến thức chuẩn bị
1.1. Bất đẳng thức qua tam thức bậc hai
Xét tam thức bậc hai f(x) = ax2 + bx+ c, a 6= 0,∆ = b2− 4ac. Ta có
các kết quả sau đây:
Định lý 1.1.1. f(x) > 0 với ∀x khi và chỉ khi
{
a > 0
∆ < 0.
Định lý 1.1.2. f(x) > 0 với ∀x khi và chỉ khi
{
a > 0
∆ 6 0.
Định lý 1.1.3. f(x) < 0 với ∀x khi và chỉ khi
{
a < 0
∆ < 0.
Định lý 1.1.4. f(x) 6 0 với ∀x khi và chỉ khi
{
a < 0
∆ 6 0.
Định lý 1.1.5. f(x) = 0 có nghiệm x1, x2 khi và chỉ khi ∆ > 0. Khi
đó: f(x) = a(x−x1)(x−x2) và
 x1 + x2 =
−b
a
x1x2 =
c
a
.
Thông thường ta chọn
x1 6 x2.
Định lý 1.1.6. x1 < α < x2 khi và chỉ khi af(α) < 0.
Định lý 1.1.7. α < x1 6 x2 khi và chỉ khi

af(α) > 0
∆ > 0
α <
−b
2a
.
Định lý 1.1.8. x1 6 x2 < α khi và chỉ khi

af(α) > 0
∆ > 0
α >
−b
2a
.
51.2. Bất đẳng thức Jensen
Mục này trình bày Bất đẳng thức Jensen. Nó sẽ được sử dụng để
chứng minh một số bất đẳng thức trong tam giác. Trước tiên ta chứng
minh bất đẳng thức này.
Định nghĩa 1.2.1. Hàm số y = f(x) được gọi là hàm lồi, (xuống phía
dưới), trong khoảng (a; b) nếu với mọi a < x1, x2 < b và mọi α ∈ (0; 1)
luôn có bất đẳng thức:
αf(x1) + (1− α)f(x2) > f
(
αx1 + (1− α)x2
)
.
Định nghĩa 1.2.2. Hàm số y = f(x) được gọi là hàm lõm, (lên phía
trên), trong khoảng (a; b) nếu với mọi a < x1, x2 < b và mọi α ∈ (0; 1)
luôn có bất đẳng thức:
αf(x1) + (1− α)f(x2) 6 f
(
αx1 + (1− α)x2
)
.
Mệnh đề 1.2.1. Giả sử y = f(x) xác định và liên tục trong (a; b) với a <
b. Hàm y = f(x) là lồi trong khoảng (a; b) khi và chỉ khi
f(x)− f(x1)
x− x1 6
f(x2)− f(x)
x2 − x hoặc
∣∣∣∣∣∣
1 x1 f(x1)
1 x f(x)
1 x2 f(x2)
∣∣∣∣∣∣ > 0 với mọi x1, x, x2 ∈ (a; b) thỏa
mãn x1 < x < x2.
Chứng minh. Giả sử y = f(x) là hàm lồi trong khoảng (a; b). Với
x1, x, x2 ∈ (a; b), x1 < x < x2, có biểu diễn:
x =
x2 − x
x2 − x1x1 +
x− x1
x2 − x1x2, f(x) 6
x2 − x
x2 − x1f(x1) +
x− x1
x2 − x1f(x2).
Như vậy có bất đẳng thức (x2−x)f(x1)+(x1−x2)f(x)+(x−x1)f(x2) > 0
hay biểu diễn dạng
∣∣∣∣∣∣
1 x1 f(x1)
1 x f(x)
1 x2 f(x2)
∣∣∣∣∣∣ > 0. Điều ngược lại là hiển nhiên.
Mệnh đề 1.2.2. Giả sử y = f(x) xác định và liên tục trong khoảng
(a; b) và có đạo hàm hữu hạn f ′(x). Khi đó y = f(x) là hàm lồi nếu và
chỉ nếu f ′(x) là hàm không giảm trong (a; b).
6Chứng minh. Giả sử y = f(x) là hàm lồi trong khoảng (a; b). Với
x1, x, x2 ∈ (a; b), x1 < x < x2, có hai biểu diễn sau đây: x =
x2 − x
x2 − x1x1 +
x− x1
x2 − x1x2 và
f(x)− f(x1)
x− x1 6
f(x2)− f(x)
x2 − x . Khi đó f
′(x1) =
lim
x→x1
f(x)− f(x1)
x− x1 6
f(x2)− f(x1)
x2 − x1 6 limx→x2
f(x2)− f(x)
x2 − x = f
′(x2). Như
vậyf ′(x1) 6 f ′(x2). Ngược lại, giả thiết f ′(x) là hàm không giảm trong
(a; b). Với x1, x, x2 ∈ (a; b), x1 < x < x2 ta có f(x)− f(x1)
x− x1 = f
′(α) và
f(x2)− f(x)
x2 − x = f
′(β), trong đó x1 < α < x < β < x2. Vì f ′(α) 6 f ′(β)
suy ra
f(x)− f(x1)
x− x1 6
f(x2)− f(x)
x2 − x . Vậy y = f(x) là hàm lồi theo Mệnh
đề 1.2.1.
Từ Mệnh đề 1.2.2 suy ra ngay kết quả dưới đây:
Định lý 1.2.1. Giả thiết y = f(x) xác định và liên tục trong khoảng
(a; b). Giả sử f(x) có đạo hàm f ′(x) cũng liên tục và có f”(x) hữu hạn
trong khoảng (a; b). Khi đó y = f(x) là hàm lồi nếu và chỉ nếu f”(x) > 0
trong (a; b).
Định lý 1.2.2. [Jensen] Nếu y = f(x) là hàm lồi trong khoảng (a; b) thì
với mọi a1, . . . , an ∈ (a; b) và mọi số thực α1, . . . , αn > 0,
n∑
k=1
αk = 1, n >
2, ta luôn có bất đẳng thức dưới đây: α1f(a1)+α2f(a2)+· · ·+αnf(an) >
f(α1a1 + α2a2 + · · ·+ αnan).
Chứng minh. Quy nạp theo n. Với n = 2 kết luận hiển nhiên đúng
theo định nghĩa. Giả sử kết luận đã đúng cho n > 2. Xét n + 1 điểm
a1, . . . , an, an+1 ∈ (a; b) và các số thực α1, . . . , αn, αn+1 > 0,
n+1∑
k=1
αk = 1
và αn+1 > 0. Đặt bn =
αn
αn + αn+1
an +
αn+1
αn + αn+1
an+1 ∈ (a; b). Theo giả
thiết quy nạp ta có:
f(α1a1 + α2a2 + · · ·+ αn−1an−1 + αnan + αn+1an+1)
= f(α1a1 + α2a2 + · · ·+ αn−1an−1 + (αn + αn+1)bn)
> α1f(a1) + α2f(a2) + · · ·+ αn−1f(an−1) + (αn + αn+1)f(bn).
7Vì f(bn) = f(
αn
αn + αn+1
an+
αn+1
αn + αn+1
an+1) >
αnf(an)
αn + αn+1
+
αn+1f(an+1)
αn + αn+1
nên
n+1∑
k=1
αkf(ak) >
n+1∑
k=1
f(αkak). Như vậy định lý đã được chứng minh.
Chú ý: Đối với các hàm số lõm ta có dấu bất đẳng thức ngược lại.
Ví dụ 1.2.1. Giả thiết số nguyên n > 2. Chứng minh bất đẳng thức:
n∏
k=1
3k − 1
3k−1
6
(
3− 3
2n
+
3
2n.3n
)n
.
Bài giải. Vì f(x) = lnx, x > 0, là hàm lồi nên theo định lý 1.2.2 có
1
n
(
n∑
k=1
ln
3k − 1
3k−1
)
6 ln
[
1
n
(
n∑
k=1
ln
3k − 1
3k−1
)]
= ln
(
3− 3
2n
+
3
2.3n
)
. Từ
đây suy ra bất đẳng thức
n∏
k=1
3k − 1
3k−1
6
(
3− 3
2n
+
3
2n.3n
)n
.
Hệ quả 1.2.1. Với a1, . . . , an, b1, . . . , bn, α1, . . . , αn > 0,
n∑
k=1
αk = 1 và
n > 2, ta luôn có các bất đẳng thức dưới đây:
(i)
n∑
k=1
αkak >
n∏
k=1
aαkk .
(ii)
n∏
k=1
(ak + bk)
αk >
n∏
k=1
aαkk +
n∏
k=1
bαkk .
(iii)
m∏
k=1
( m∑
j=1
akj
)αk
>
m∑
j=1
n∏
k=1
aαkkj với mọi akj > 0.
(iv) [Cauchy]
n∑
k=1
ak > n n
√
n∏
k=1
ak.
Chứng minh. (i) Xét hàm lõm f(x) = lnx. Theo Định lý Jensen ta có
ln
( n∏
k=1
aαkk
)
=
n∑
k=1
αk ln ak 6 ln
( n∑
k=1
αkak
)
. Do
n∏
k=1
aαkk 6
n∑
k=1
αkak.
(ii) Do
n∑
k=1
αk
ak
ak + bk
>
n∏
k=1
( ak
ak + bk
)αk
,
n∑
k=1
αk
bk
ak + bk
>
n∏
k=1
( bk
ak + bk
)αk
theo (i) nên sau khi cộng hai vế được 1 >
n∏
k=1
( ak
ak + bk
)αk
+
8n∏
k=1
( bk
ak + bk
)αk
. Qua quy đồng ta nhận được bất đẳng thức
n∏
k=1
(ak +
bk)
αk >
n∏
k=1
aαkk +
n∏
k=1
bαkk .
(iii) Sử dụng (ii) để quy nạp theo m sẽ được (iii). Với α1 = · · · = αn = 1
n
,
từ (i) suy ra (iv).
1.3. Bất đẳng thức Karamata, Schur, Muirheard
Bộ n số thực (a) = (a1, a2, . . . , an) thỏa mãn a1 > a2 > · · · > an được
gọi là bộ số không tăng. Đặt |(a)| = a1 + a2 + · · · + an. Trong tập hợp
tất cả các bộ số không tăng A = {(a) = (ak)} ta định nghĩa quan hệ thứ
tự: Giả sử (a) = (ak), (b) = (bk) là hai bộ số không tăng. Định nghĩa
(a) > (b) khi và chỉ khi a1 + · · ·+ ak > b1 + · · ·+ bk,∀ k = 1, 2, . . . , n;
Còn nếu có k để a1 + · · ·+ ak > b1 + · · ·+ bk thì ta viết (a) > (b).
Định nghĩa 1.3.1. Giả sử có hai bộ số không tăng (a) = (a1, a2, . . . , an)
và (b) = (b1, b2, . . . , bn). Bộ (a) được gọi là trội hơn (b) nếu các điều kiện
sau đây được thỏa mãn:
{
a1 + · · ·+ ak > b1 + · · ·+ bk
k = 1, 2, . . . , n− 1; |(a)| = |(b)|.
Mệnh đề 1.3.1. [Karamata] Giả sử y = f(x) là một hàm lồi trên
khoảng (a; b) và các bộ không tăng (a), (b) với ak, bk ∈ (a; b) với k =
1, 2, . . . , n. Nếu bộ (a) trội hơn bộ (b) thì có bất đẳng thức
n∑
k=1
f(ak) >
n∑
k=1
f(bk); còn khi y = f(x) là hàm lõm thì
n∑
k=1
f(ak) 6
n∑
k=1
f(bk).
Chứng minh: Đặt ck = δf(ak, bk) =
f(bk)− f(ak)
bk − ak , k = 1, 2, . . . , n. Theo
Mệnh đề 1.1.1, dãy (ck) là dãy đơn điệu giảm bởi vì (a) và (b) là dãy
không tăng. Đặt Ak =
k∑
i=1
ai, Bk =
k∑
i=1
bi với A0 = B0 = 0, k = 1, 2, . . . , n.
9Từ |(a)| = |(b)| suy ra An = Bn. Biến đổi hiệu
H =
n∑
k=1
f(ak)−
n∑
k=1
f(bk) =
n∑
k=1
(
f(ak)− f(bk)
)
=
n∑
k=1
ck(ak − bk)
=
n∑
k=1
ck(Ak − Ak−1 −Bk +Bk−1)
=
n∑
k=1
ck(Ak −Bk)−
n∑
k=1
ck(Ak−1 −Bk−1)
=
n−1∑
k=1
ck(Ak −Bk)−
n−1∑
k=0
ck+1(Ak −Bk) =
n−1∑
k=1
(ck − ck+1)(Ak −Bk).
Vì Ak > Bk và ck 6 ck+1 với mọi k = 1, 2, . . . , n. Vậy H 6 0.
Ví dụ 1.3.1. Với các số thực dương a, b, c luôn có bất đẳng thức
1
a+ b
+
1
b+ c
+
1
c+ d
+
1
d+ a
6 1
2a
+
1
2b
+
1
2c
+
1
2d
.
Bài giải. Không hạn chế có thể giả thiết a > b > c > d > 0. Khi đó
có

2a > a+ b
2a+ 2b > a+ b+ b+ c
2a+ 2b+ 2c > a+ b+ b+ c+ c+ d
2a+ 2b+ 2c+ 2d = a+ b+ b+ c+ c+ d+ d+ a.
Từ đây suy ra
( 2a, 2b, 2c, 2d ) trội hơn bộ (a+ b, b+c, c+d, d+a). Vì y =
1
x
với x > 0
là hàm lồi nên ta có
1
a+ b
+
1
b+ c
+
1
c+ d
+
1
d+ a
6 1
2a
+
1
2b
+
1
2c
+
1
2d
.
Theo mệnh đề ( 1.3.1).
Ví dụ 1.3.2. Chứng minh rằng với ba số thực x, y, z ∈ [−1; 1] thỏa mãn
x+ y + z =
1
2
ta luôn có bất đẳng thức x2012 + y2012 + z2012 6 2 + 1
22012
.
Bài giải. Vì x, y, z bình đẳng nên có thể cho 1 > x > y > z > −1.
Hàm y = x2012 là hàm lồi ( xuống phía dưới ) vì f”(x) > 0 trong (−1; 1)
theo định lý ( 1.2.1 ). Ta xây dựng bộ trội của ( x, y, z ) như sau:{
1 > x
x+ y = −z + 1
2
6 1
2
.
Vậy (1,
1
2
,−1) là một bộ trội của ( x, y, z ).
Theo mệnh đề ( 1.3.1). Ta có x2012 + y2012 + z2012 6 2 + 1
22012
.
10
Mệnh đề 1.3.2. [Schur] Với α, β > 0 có S(α+2β,0,0) + S(α,β,β) >
2S(α+β,β,0).
Chứng minh. Bất đẳng thức cần chứng minh chính là bất đẳng thức
dưới đây: aα+2β + bα+2β + cα+2β + aαbβcβ + aβbαcβ) + aβbβcα > aα+βbβ +
aα+βcβ + bα+βaβ + bα+βcβ + cα+βaβ + cα+βbβ, (∗), với a, b, c > 0. Không
hạn chế có thể giả thiết a > b > c > 0. Viết lại bất đẳng thức (*) thành
aα(aβ − bβ)(aβ − cβ) + bα(bβ − cβ)(bβ − aβ) + cα(cβ − aβ)(cβ − bβ) > 0.
Vì cα(cβ − aβ)(cβ − bβ) > 0 nên chỉ cần chứng minh aα(aβ − bβ)(aβ −
cβ) + bα(bβ − cβ)(bβ − aβ) > 0, hay aα(aβ − cβ)− bα(bβ − cβ) > 0, nhưng
bất đẳng thức này là hiển nhiên.
Ví dụ 1.3.3. Giả sử a, b, c là độ dài ba cạnh một tam giác. Chứng minh
rằng: a3 + b3 + c3 + 3abc > a2b+ a2c+ b2c+ b2a+ c2a+ c2b.
Bài giải. Bất đẳng thức có được từ Mệnh đề 1.3.2 với α = β = 1.
Bổ đề 1.3.1. Nếu β = Tij(α) thì có bất đẳng thức Mβ(a) > Mα(a).
Dấu bằng xảy ra khi a1 = · · · = an.
Chứng minh. Với mỗi cặp chỉ số h, k, h < k, hiệuMβ(a)−Mα(a) chứa số
hạng dạng: B = A(aβih a
βj
k + a
βj
h a
βi
k − aαih aαjk − aαjh aαik ) với A > 0. Biến đổi
hiệu B = A(aαi+1h a
αj−1
k +a
αj−1
h a
αi+1
k −aαih aαjk −aαjh aαik ) = Aaαj−1h aαj−1k (ah−
ak)(a
αi−αj+1
h − aαi−αj+1k ) > 0. Do vậy Mβ(a) > Mα(a). Dấu bằng xảy ra
khi từng B = 0 hay ah = ak với mọi h, k = 1, 2, . . . = n.
Bổ đề 1.3.2. Nếu (β) > (α), (β) 6= (α) và |(α)| = |(β)|, thì sau một số
hữu hạn phép biến đổi tuyến tính Tij sẽ chuyển (α) thành (β).
Chứng minh. Vì (β) 6= (α) nên có chỉ số i nhỏ nhất để βi 6= αi. Do bởi
(β) > (α) nên βi > αi. Từ |(α)| = |(β)| suy ra
n∑
k=1
(βk − αk) = 0. Do
βi > αi nên tồn tại j để 0 6 βj < αj và i < j. Tác động Tij vào (α) ta
nhận được γ = Tij(α) với γi = αi + 1, γj = αj − 1, còn γk = αk với mọi
k 6= i, j. Như vậy |βi − αi| = |βi − γi + 1| = |βi − γi| + 1 và |βj − αj| =
|βj − γj − 1| = |βj − γj| + 1. Từ hai hệ thức này suy ra
n∑
k=1
|βk − γk| =
11
n∑
k=1
|βk − αk| − 2. Do vậy, khi tác động Tij làm tổng
n∑
k=1
|βk − αk| giảm
được 2 đơn vị, sau một số hữu hạn bước, ta có
n∑
k=1
|βk − δk| = 0 hay đã
chuyển được (α) thành (β).
Từ hai bổ đề trên ta suy ra ngay Bất đẳng thức Muirheard dưới đây:
Mệnh đề 1.3.3. [Muirheard] Với các số dương a1, a2, . . . , an, xảy ra
bất đẳng thức Mα(a) > Mβ(a) khi và chỉ khi α > β và |α| = |β|. Dấu
bằng chỉ xảy ra khi α = β và a1 = a2 = · · · = an.
Chú ý rằng, khi vận dụng Bất đẳng thức Muirheard ta phải chọn bộ
trội thế nào để nhanh có kết quả.
Ví dụ 1.3.4. Với ba số thực dương a, b, c chúng ta có bất đẳng thức:
1
a3 + b3 + abc
+
1
b3 + c3 + abc
+
1
c3 + a3 + abc
6 1
abc
.
Chứng minh. Bất đẳng thức tương đương với:
abc[
(
a3 + b3 + abc
) (
b3 + c3 + abc
)
+
(
b3 + c3 + abc
) (
c3 + a3 + abc
)
+(
c3 + a3 + abc
) (
a3 + b3 + abc
)
]. Hay a6b3 + a6c3 + b6c3 + b6a3 + c6a3 +
c6b3 > 2
[
a5b2c2 + a2b5c2 + a2b2c5
]
. Bất đẳng thức này tương đương với
bất đẳng thức M(6,3,0) (a) >M(5,2,2) (a) .
Ví dụ 1.3.5. Với ba số thực dương a, b, c, ta luôn có bất đẳng thức:
a3 + b3 + c3 > (a+ b) (b+ c) (c+ a) .
Chứng minh. Bất đẳng thức cần chứng minh tương đương với abc >
(a+ b− c) (b+ c− a) (c+ a− b) . Nếu vế phải có một hoặc ba thừa số
âm thì bất đẳng thức hiển nhiên đúng. Nếu vế phải chỉ có hai thừa số âm,
chẳng hạn b+ c− a < 0, a+ b− c < 0 thì 2b = b+ c− a+ a+ b− c < 0:
Mâu thuẫn. Ta chỉ cần xét trường hợp cả ba thừa số đều không âm.
Đặt x = b + c − a, y = c + a − b, z = a + b − c > 0. Khi đó 2a =
y + z, 2b = z + x, 2c = x + y. Vậy abc =
1
8
(x+ y) (y + z) (z + x) >
xyz = (a+ b− c) (b+ c− a) (c+ a− b) .
12
1.4. Một vài hàm tự chọn
Khảo sát hàm sốf(x) = xα − αx với các kết quả sau đây:
Mệnh đề 1.4.1. Với x > 0 ta có các bất đẳng thức sau:
(i) xα > αx+ 1− α khi α > 1.
(ii) xα 6 αx+ 1− α khi 0 < α < 1.
Mệnh đề 1.4.2. [Bernoulli]. Nếu x > −1 thì (1 + x)n > 1 + nx với
mọi n = 0, 1, 2, ....Dấu bằng chỉ xảy ra khi x = −1 hoặc n = 1. Tổng
quát, nếu α > 1 và x > −1 thì (1 + x)α > 1 + αx.
Chứng minh. Với n = 1 ta có đẳng thức xảy ra. Giả sử bất đẳng thức
đúng đến n = k > 1 ta sẽ đi chứng minh bất đẳng thức đúng đến n =
k+ 1. Thật vậy ta có (1 + x)k+1 = (1 + x)k (1 + x) > (1 + kx) (1 + x) =
1 + kx+ x+ kx2 > 1 + (k + 1)x.
Ví dụ 1.4.1. Chứng minh rằng, nếu α1, α2, α3 > 0 và α1 + α2 + α3 = 1
thì với mọi số thực u1, u2, u3 > 0 có bất đẳng thức
uα11 u
α2
2 u
α3
3 6 α1u1 + α2u2 + α3u3.
Bài giải. Từ xα 6 αx + 1 − α khi 0 < α < 1, theo Mệnh đề ( 1.4.1 )
và thay x =
p
q
ta được pαq1−α 6 αp + (1− α) q, trong đó p, q > 0. Với
β = 1−α và α, β > 0, α+ β = 1, có pαqβ 6 αp+ βq. Với p = u1, α = α1
và q = uα22 u
α3
3 , α2 + α3 = β, được u
α1
1 u
α2
2 u
α3
3 6 α1u1 + βu
α2
β
2 u
α3
β
3 . Vì
u
α2
β
2 u
α3
β
3 6
α2
β
u2 +
α3
β
u3 nên u
α1
1 u
α2
2 u
α3
3 6 α1u1 + α2u2 + α3u3.
Ví dụ 1.4.2. Với x ∈ (0, pi
2
) chứng minh rằng sinx+ tanx > 2x.
Chứng minh. Xét hàm số f(x) = sinx + tanx − 2x với 0 < x < pi
2
có
f ′(x) = cosx +
1
cos2x
− 2 > 2
√
cosx.
1
cos2x
− 2 > 0, chúng ta suy ra
f(x) > f(0) = 0 hay sinx+ tanx− 2x > 0. Vậy sinx+ tanx > 2x.
13
Ví dụ 1.4.3. Cho các số a, b, c thỏa mãn
{
a+ b+ c = 8
ab+ bc+ ca = 16
. Chứng
minh rằng 0 6 a 6 16
3
.
Chứng minh. Từ giả thiết ta suy ra
{
b+ c = 8− a
bc = 16− a(8− a) = a2 − 8a+ 16,
nên b và c là nghiệm của phương trình ẩn t: t2−(8−a)t+(a2−8a+16) = 0.
Vì phương trình này có nghiệm nên ∆ = (8− a)2 − 4(a2 − 8a+ 16) > 0
hay 0 6 a 6 16
3
. Tương tự ta có 0 6 b 6 16
3
và 0 6 c 6 16
3
.
Hoàn toàn tương tự ta có bài toán sau: Cho a, b, c là ba số thỏa mãn
hệ
{
a+ b+ c = 10
ab+ bc+ ca = 25.
Khi đó 0 6 a, b, c 6 20
3
.
14
Chương 2
Một số đồng nhất thức và bất đẳng
thức trong tam giác
Mục này tập trung trình bày một phương pháp phát hiện ra các đồng
nhất thức và bất đẳng thức trong tam giác qua phương trình đa thức
bậc ba.
2.1. Đa thức bậc ba liên quan đến tam giác.
Cho ∆ABC với độ dài ba cạnh là a, b, c; bán kính các đường tròn
nội, ngoại tiếp là r, R; bán kính đường tròn bàng tiếp là r1, r2, r3; nửa
chu vi p và diện tích S. Ta sẽ chỉ ra a, b, c là ba nghiệm của x3 − 2px2 +(
p2 + r2 + 4Rr
)
x−4Rrp = 0 và r1, r2, r3 là ba nghiệm của phương trình
x3 − (4R + r)x2 + p2x− p2r = 0.
Mệnh đề 2.1.1. Cho ∆ABC với độ dài cạnh BC = a, CA = b, AB = c.
Ký hiệu p là nửa chu vi; r và R là bán kính các đường tròn nội, ngoại
tiếp. Khi đó a, b, c là ba nghiệm của phương trình dưới đây:
x3 − 2px2 + (p2 + r2 + 4Rr)x− 4Rrp = 0.
Chứng minh. Từ tan
A
2
=
r
p− a và a = 2R sinA ta suy ra hệ thức
a = 2R
2 tan
A
2
1 + tan2
A
2
hay a = 4R
r
p− a
1 +
(
r
p− a
)2 = 4Rr p− ar2 + (p− a)2 .
Như vậy, ta có quan hệ a
(
a2 − 2pa+ p2 + r2) = 4Rr (p− a) hay a3 −
2pa2 +
(
p2 + r2 + 4Rr
)
a−4Rrp = 0. Do đó a là một nghiệm của phương
trình x3 − 2px2 + (p2 + r2 + 4Rr)x− 4Rrp = 0.
Tương tự, b và c cũng là nghiệm của phương trình trên.
Mệnh đề 2.1.2. Cho ∆ABC với độ dài cạnh BC = a, CA = b, AB = c.
Ký hiệu p là nửa chu vi; r và R là bán kính các đường tròn nội, ngoại
15
tiếp. Khi đó sinA, sinB, sinC là ba nghiệm của phương trình dưới đây:
x3 − p
R
x2 +
p2 + r2 + 4Rr
4R2
x− rp
2R2
= 0.
Chứng minh. Vì a = 2R sinA và a lại là nghiệm của phương trình x3 −
2px2 +
(
p2 + r2 + 4Rr
)
x− 4Rrp = 0 nên thay a vào phương trình trên,
ta có 8R3sin3A− 8pR2sin2A+ (p2 + r2 + 4Rr) 2R sinA− 4Rrp = 0 hay
sin3A− p
R
sin2A+
p2 + r2 + 4Rr
4R2
sinA− rp
2R2
= 0. Như vậy sinA là một
nghiệm của phương trình x3 − p
R
x2 +
p2 + r2 + 4Rr
4R2
x− rp
2R2
= 0.
Tương tự, sinB và sinC cũng là nghiệm của phương trình trên.
Mệnh đề 2.1.3. Cho ∆ABC với diện tích S và độ dài bán kính các
đường tròn nội, ngoại tiếp là r, R. Gọi ha, hb, hc là độ dài ba đường cao.
Khi đó ha, hb, hc là ba nghiệm của phương trình bậc ba sau:
y3 − S
2 + 4Rr3 + r4
2Rr2
y2 +
2S
Rr
y − 2S
2
R
= 0.
Đặt Hn = h
n
a + h
n
b + h
n
c với n=1,2,...Khi đó ta có hệ thức liên hệ sau:
Hn+3 − S
2 + 4Rr3 + r4
2Rr2
Hn+2 +
2S
Rr
Hn+1 − 2S
2
R
Hn = 0.
Bài giải. Vì a, b, c là ba nghiệm của phương trình bậc ba x3 − 2px2 +(
p2 + r2 + 4Rr
)
x − 4Rrp = 0 nên 2S
a
,
2S
b
,
2S
c
là nghiệm của phương
trình 2S2 − 2S
r
y +
S2
r2
+ 4Rr + r2
2
y2 − Ry3 = 0. Do vậy ha, hb, hc là
ba nghiệm của y3 − S
2 + 4Rr3 + r4
2Rr2
y2 +
2S
Rr
y − 2S
2
R
= 0. Vậy Hn+3 −
S2 + 4Rr3 + r4
2Rr2
Hn+2 +
2S
Rr
Hn+1 − 2S
2
R
Hn = 0.
Mệnh đề 2.1.4. Cho ∆ABC với nửa chu vi p; bán kính các đường tròn
nội, ngoại tiếp là r, R; bán kính các đường tròn bàng tiếp là r1, r2, r3. Khi
đó r1, r2, r3 là nghiệm của phương trình x
3− (4R + r)x2 +p2x−p2r = 0.
16
Chứng minh. Từ tan
A
2
=
r1
p
và a = 2R sinA suy ra a = 2R
2 tan
A
2
1 + tan2
A
2
.
Vậy a = 4R
r1
p
1 +
r21
p2
= 4Rr1
p
r21 + p
2
. Bởi vì r1 (p− a) = S = pr nên
chúng ta có quan hệ sau:
(r1 − r) p
r1
= a = 4Rr1
p
r21 + p
2
hay tương đương
(r1 − r)
(
r21 + p
2
)
= 4Rr21. Do vậy r1 là nghiệm của phương trình x
3 −
(4R + r)x2 + p2x− p2r = 0.
Tương tự, r2, r3 cũng là nghiệm của phương trình này.
Mệnh đề 2.1.5. Cho ∆ABC với nửa chu vi p; bán kính các đường tròn
nội, ngoại tiếp là r, R. Khi đó tan
A
2
, tan
B
2
, tan
C
2
là ba nghiệm của
phương trình x3 − 4R + r
p
x2 + x− r
p
= 0. Từ đây ta suy ra
(i) tan
A
2
+ tan
B
2
+ tan
C
2
=
4R + r
p
.
(ii) tan
A
2
tan
B
2
+ tan
B
2
tan
C
2
+ tan
C
2
tan
A
2
= 1. Từ đây chúng
ta suy ra hai bất đẳng thức tan
A
2
+ tan
B
2
+ tan
C
2
>
√
3;
tan
A
2
tan
B
2
tan
C
2
6 1
3
√
3
.
(iii) tan
A
2
tan
B
2
tan
C
2
=
r
p
.
Chứng minh. Vì r1 = p tan
A
2
và r1 lại là nghiệm của phương trình x
3−
(4R + r)x2+p2x−p2r = 0 nên p3tan3A
2
−(4R + r) p2tan2A
2
+p2p tan
A
2
−
p2r = 0 hay tan3
A
2
− 4R + r
p
tan2
A
2
+ tan
A
2
− r
p
= 0. Như vậy tan
A
2
là
một nghiệm của x3 − 4R + r
p
x2 + x− r
p
= 0. Tương tự tan
B
2
và tan
C
2
17
cũng là nghiệm của phương trình này. Các kết quả (i), (ii), (iii) được suy
ra từ Định lý Viét.
Mệnh đề 2.1.6. Cho ∆ABC với nửa chu vi p; bán kính các đường tròn
nội, ngoại tiếp là r, R. Khi đó ta có:
(i) cosA, cosB, cosC là ba nghiệm của phương trình đa thức dưới đây:
x3 − R + r
R
x2 +
−4R2 + r2 + p2
4R2
x− p
2 − (2R + r)2
4R2
= 0.
(ii) cosA+ cosB + cosC = 1 +
r
R
.
(iii) p = (1− cos

File đính kèm:

  • pdfDong nhat thuc va bat dang thuc trong tam giac.pdf